6
Taller Análisis de Presiones Juan Camilo Posada G., Código 2010191469 Nelson Adolfer Durán R., Código 2006135792 Jorge Armando Arenas P., Código 20122113303 Carlos Andrés Pérez M., Código 20122111982 Universidad Surcolombiana 1. Solución: Se tiene que q = 250 STB/D k = 25 mD c o = 4 × 10 -6 psi -1 r w = 3 in = 0.25 ft μ = 0.75 cP φ = 0.20 c f = 2 × 10 -6 psi -1 h = 60 ft B = 1.25 bbl/STB a) La aproximación logarítmica E i (-x ) ln (1.781 x ) es válida solamente para valores de x < 0.0025, esto es, para cuando 948φμc t r 2 / (kt ) < 0.0025. Ahora, si consideramos que r = r w y que c t = c o + c f tenemos que el valor límite para el tiempo cumple la relación 0.0025 = 948 (0.20)(0.75) 4 × 10 -6 + 2 × 10 -6 (0.25) 2 25 t t = 8.532 × 10 -4 horas 3.07 seg. b ) Para t = 30 h se tiene que x = 948 (0.20)(0.75) 4 × 10 -6 + 2 × 10 -6 (0.25) 2 (25)(30) = 7.11 × 10 -8 y por tanto E i (-x )= ln (1.781 x )= ln 1.781 · 7.11 × 10 -8 = -15.882, y de este modo P D = - 1 2 E i (-x )= - 1 2 (-15.882)= 7.941. Ahora procedemos a calcular el valor de ΔP como sigue P D = kh 141.2q μB P i - P wf , ΔP = 141.2q μB kh P D = 141.2 (250)(0.75)(1.25) (25)(60) (7.941)= 175.20 psi NOTA: Dado que no se posee P i entonces lo que se entrega en este caso es ΔP . c ) El gradiente de presión con respecto al radio para un t dado corresponde a P ( r , t ) r = dP dr t = ım Δr 0 ΔP Δr t = ım Δr 0 P i - 70.6 qB μ kh E i - 948φμc t ( r + Δr ) 2 kt - P i - 70.6 qB μ kh E i - 948φμc t r 2 kt Δr = ım Δr 0 70.6qB μ kh Δr E i - 948φμc t r 2 kt - E i - 948φμc t ( r + Δr ) 2 kt Para r = 1500 ft y t = 30 h, y considerando los valores dados de los parámetros para el ejercicio se tendría que P ( r , t ) r (1500,30) = ım Δr 0 70.6qB μ kh Δr E i - 948φμc t r 2 kt - E i - 948φμc t ( r + Δr ) 2 kt = 11.03125 l´ ım Δr 0 E i (-2.5596) - E i -1.1376 × 10 -6 (1500 + Δr ) 2 Δr 1

Tarea01Resuelta

Embed Size (px)

DESCRIPTION

Tarea01Resuelta

Citation preview

Page 1: Tarea01Resuelta

TallerAnálisis de Presiones

Juan Camilo Posada G., Código 2010191469 Nelson Adolfer Durán R., Código 2006135792Jorge Armando Arenas P., Código 20122113303 Carlos Andrés Pérez M., Código 20122111982

Universidad Surcolombiana

1. Solución: Se tiene que

q = 250 STB/D

k = 25 mD

co = 4× 10−6 psi−1

rw = 3 in= 0.25 ft

µ= 0.75 cP

φ= 0.20

c f = 2× 10−6 psi−1

h = 60 ft

B = 1.25 bbl/STB

a) La aproximación logarítmica Ei (−x) ≈ ln (1.781x) es válida solamente para valores de x < 0.0025, esto es, paracuando 948φµct r 2/ (kt )< 0.0025. Ahora, si consideramos que r = rw y que ct = co + c f tenemos que el valorlímite para el tiempo cumple la relación

0.0025=948 (0.20) (0.75)

4× 10−6+ 2× 10−6�

(0.25)2

25t⇒ t = 8.532× 10−4 horas≈ 3.07 seg.

b ) Para t = 30 h se tiene que

x =948 (0.20) (0.75)

4× 10−6+ 2× 10−6�

(0.25)2

(25) (30)= 7.11× 10−8

y por tanto Ei (−x) = ln (1.781x) = ln�

1.781 · 7.11× 10−8�

= −15.882, y de este modo PD = −12 Ei (−x) =

−12 (−15.882) = 7.941. Ahora procedemos a calcular el valor de∆P como sigue

PD =kh

141.2qµB

Pi − Pw f

, ∆P =141.2qµB

khPD =

141.2 (250) (0.75) (1.25)(25) (60)

(7.941) = 175.20 psi

NOTA: Dado que no se posee Pi entonces lo que se entrega en este caso es∆P .c) El gradiente de presión con respecto al radio para un t dado corresponde a

∂ P (r, t )

∂ r=

d P

d r

t=

lım∆r→0

∆P

∆r

t

= lım∆r→0

Pi − 70.6qBµ

khEi

−948φµct (r +∆r )2

kt

��

Pi − 70.6qBµ

khEi

−948φµct r 2

kt

��

∆r

= lım∆r→0

70.6qBµ

kh∆r

Ei

−948φµct r 2

kt

− Ei

−948φµct (r +∆r )2

kt

��

Para r = 1500 ft y t = 30 h, y considerando los valores dados de los parámetros para el ejercicio se tendría que

∂ P (r, t )

∂ r

(1500,30)= lım∆r→0

70.6qBµ

kh∆r

Ei

−948φµct r 2

kt

− Ei

−948φµct (r +∆r )2

kt

��

= 11.03125 lım∆r→0

Ei (−2.5596)− Ei

−1.1376× 10−6 (1500+∆r )2�

∆r

1

Page 2: Tarea01Resuelta

Si asumimos un ∆r pequeño (por ejemplo de 0.01 ft) puede obtenerse una aproximación muy satisfactoria delgradiente de presión (derivación numérica), y más considerando el orden de magnitud de las distancias manejadaspara yacimientos.Al reemplazar y operar se obtiene que el gradiente de presión en ese punto del yacimiento y para el instantepedido corresponde a 1.1375× 10−3 psi/ft.NOTA: Otra alternativa es usando la regla de Leibnitz (derivada de una integral), quedando por tanto que

∂ P (r, t )

∂ r=∂

∂ r

Pi − 70.6qBµ

khEi

−948φµct r 2

kt

��

=−70.6qBµ

kh

∂ r(Ei (−x)) =−70.6

qBµ

kh

d

d x(Ei (−x))

d x

d t

=−70.6qBµ

kh

d

d x

�ˆ

x

e−u

ud u�

d x

d t=−70.6

qBµ

kh

d

d x

lımu→∞

e−u

u−

e−x

x

1896φµct r

k t

=−70.6qBµ

kh

d

d x

−e−x

x

1896φµct r

k t

= 133857.6�

qBµ

kh

φµct r

k t

� exp�

−948φµct r 2

kt

948φµct r 2

kt

= 141.2qBµ

kh rexp

−948φµct r 2

kt

Al reemplazar con los valores dados tenemos que

∂ P (r, t )

∂ r

(1500,30)= 141.2

(250) (1.25) (0.75)(25) (60) (1500)

exp

−948 (0.20) (0.75)

6× 10−6�

(1500)2

(25) (30)

= 1.1375× 10−3 psi/ft

(Como se observa el resultado es idéntico al de la derivada numérica)

2. Solución: La información proporcionada corresponde a

q = 150 STB/DPw f = 1500 psiPpromedio = 2500 psi

h = 35 ftre = 1000 ftrw = 0.25 ft

µ= 0.8 cP

B = 1.25 RB/STB

a) Sabemos que P = 12

Pw f + Pi

y por tanto Pi = 2P + Pw f = 2 (2500)− 1500= 3500 psi.

b ) De la ley de Darcy para flujo radial tenemos que

q =7.08kh

Pi − Pw f

µ ln (re/rw )

y por tanto

150=7.08k (35) (3500− 1500)(0.8) ln (1000/0.25)

= 74692.1k , ⇒ k = 2.01× 10−3 D= 2.01 mD

c) Dado que la permeabilidad del núcleo estudiado fue de 50 mD, mientras que la estimada para las condicionesactuales es sólo de 2.01 mD, concluímos que el pozo se encuentra con daño. Para determinar el skin consideramosla relación

Pi − Pw f =141.2qµB

kh

ln�

re

rw

+ s

quedando que

3500− 1500=141.2 (150) (0.8) (1.25)

(50) (35)

ln�

10000.25

+ s�

2000= 12.10286 (8.29405+ s)

s = 156.96

El excesivo valor de skin se atribuye a la disminución tan drástica de la permeabilidad (unas 25 veces aprox.).

3. Solución: Se tiene que

2

Page 3: Tarea01Resuelta

q = 800 STB/D (por 20 días)

k = 25 mD

ct = 5× 10−5 psi−1

rw = 0.335 ft

µ= 0.55 cP

φ= 0.20

Pi = 3000 psia

h = 60 ft

B = 1.15 bbl/STB

Se tiene que la caída de presión en el pozo W viene dada por ∆PW = ∆PR ±∆PI (sindo «−» en el caso de un pozoinyector).

Dado que la el caudal permanece constante durante el tiempo estudiado asumiremos que la presión en la falla esconstante y por tanto el pozo imagen se considerará inyector. Tendremos entonces que

∆PW =70.6qµB

khEi

−948φµct r 2

w

kt

−70.6qµB

khEi

−948φµct (2d )2

kt

=70.6qµB

kh

Ei

−948φµct r 2

w

kt

− Ei

−948φµct (2d )2

kt

��

donde d corresponde a la distancia del pozo a la falla (d = 500 ft). De este modo la presión del pozo fluyendo equivalea

∆PW =70.6 (800) (0.55) (1.15)

(25) (60)

Ei

−948 (0.2) (0.55)

5× 10−5�

(0.335)2

(25) (20 · 24)

− Ei

−948 (0.2) (0.55)

5× 10−5�

(2 · 500)2

(25) (20 · 24)

��

= 23.81573�

Ei

4.87618× 10−8�− Ei (0.4345)

= 23.81573 (16.2591− 0.6479)

= 371.793 psi

4. Solución:

k = 125 mDct = 18× 10−6 psi−1

rw = 0.335 ft

µ= 0.65 cPφ= 0.18Pi = 2500 psia

h = 83 ft

B = 1.32 bbl/STB

Para este problema consideraremos la superposición en el tiempo, donde q1 = 800, q2 = 0, t1 = 6 · 24 = 144, y t2 =7× 24= 168.

∆PN =70.6q1µB

khEi

−948φµct r 2

kt1

+70.6 (q2− q1)µB

khEi

−948φµct r 2

k (t2− t1)

=70.6µB

kh

q1Ei

−948φµct r 2

kt1

+(q2− q1)Ei

−948φµct r 2

k (t2− t1)

��

=70.6 (0.65) (1.32)(125) (83)

(800)Ei

948 (0.18) (0.65)�

18× 10−6�

(1500)2

(125) (144)

+(0− 800)Ei

−948 (0.18) (0.65)

18× 10−6�

(1500)2

(125) (168− 144)

��

= 4.6708 (Ei (0.249561)− Ei (1.497366)) = 4.415 psi

5. Solución:

k = 50 mDµ= 0.256 cPqA= 1240 STB/DsA= 0rBC = 1320 ft

B = 1.345 RB/STBh = 24 fttA= 4 hqB = 1240 STB/DsB = 5

φ= 0.18

ct = 21× 10−6

rAC = 660 ft

tB = 16 h

3

Page 4: Tarea01Resuelta

a) La caída de presión ∆PAC se obtiene como sigue:

∆PAC =70.6qAµB

kh

Ei

948φµct r 2AC

ktA

+ 2sA

=70.6 (1240) (0.256) (1.345)

(50) (24)

Ei

−948 (0.18) (0.256)

21× 10−6�

(660)2

(50) (4)

+ 2 (0)

= 25.1193Ei (1.99801) = 1.232 psi

b ) La caída de presión ∆PBC se obtiene de la misma manera que la anterior:

∆PBC =70.6qBµB

kh

Ei

948φµct r 2BC

ktB

+ 2sB

=70.6 (1240) (0.256) (1.345)

(50) (24)

Ei

−948 (0.18) (0.256)

21× 10−6�

(1320)2

(50) (16)

+ 2 (5)

= 25.1193 (Ei (1.99801)+ 10) = 301.382 psi

c) Tenemos, por el principio de superposición, que ∆PC = ∆PAC +∆PBC = 302.613 psi. Y dado que la presiónincial del yacimiento es 2776 psi, entonces PC = 2776− 302.613= 2473.387 psi.

6. Solución: Tenemos que

qA= 700 STB/D

qB = 1400 STB/D

k = 125 mD

h = 100 ft

Pi = 5000 psi

φ= 0.25

ct = 3× 106 psi−1

µ= 1.4 cP

B = 1.36 RB/STB

a) Dado que rA+ rB = 1000 ft, si asumimos que∆PAN =∆PBN entonces

70.6qAµB

khEi

−948φµct r 2

A

kt

=70.6qBµB

khEi

−948φµct r 2

B

kt

qAEi

−948φµct r 2

A

kt

= qB Ei

−948φµct r 2

B

kt

700Ei

−948 (0.25) (1.4)

3× 10−6�

r 2A

(125) (100 · 24)

= 1400Ei

−948 (0.25) (1.4)

3× 10−6�

(1000− rA)2

(125) (100 · 24)

Ei

−3.318× 10−9 r 2A

= 2Ei

−3.318× 10−9 (1000− rA)2�

Esta ecuación es válida para cuando rA = 66.76 ft (resuelta por ensayo y error para valores dentro del intervaloestudiado).Esto es, según la gráfica, el punto N se encuentra separado del pozo A una distancia de 66.69 pies a la derecha.En este punto

∆PAN =∆PBN =70.6 (700) (1.4) (1.36)

(125) (100)Ei

−948 (0.25) (1.4)

3× 10−6�

(66.76)2

(125) (100 · 24)

= 7.52765Ei

−1.4788× 10−5�

= 79.38 psi

Por tanto ∆PN = ∆PAN +∆PBN = 158.76 psi. La presión en N corresponde entonces a PN = Pi −∆PN =5000− 158.76= 4841.24 psi.

b ) Sabemos que∆PM =∆PA−∆PB (en este caso resta por considerarse al pozo B como inyector). De este modo

∆PM =70.6qAµB

khEi

−948φµct a2

1

kt

−70.6qBµB

khEi

−948φµct b 2

1

kt

=70.6µB

kh

qAEi

−948φµct a2

1

kt

− qB Ei

−948φµct b 2

1

kt

��

4

Page 5: Tarea01Resuelta

en donde al reemplazar y operar los valores de los parámetros conocidos tendríamos que

∆PM = 0.010754�

700Ei

−3.318× 10−9a21

− 1400Ei

−3.318× 10−9b 21

��

= 7.52765�

Ei

−3.318× 10−9a21

− 2Ei

−3.318× 10−9b 21

��

Ahora, dado que PM = Pi −∆PM quedando que

PM = 5000− 7.52765�

Ei

−3.318× 10−9a21

− 2Ei

−3.318× 10−9 b 21

��

,

esto es, dependiente de las distancias a1 y b2, y donde se sobreentiende que a1+ b1 ≥ 1000 ft (desigualdad trian-gular).OBSERVACIÓN: Ahora bien, si asumimos que el punto M está relativamente cerca de los pozos A y B de modoque para ambos casos se cumpla que x < 0.0025 entonces la siguiente aproximación sería válida:

PM = 5000− 7.52765�

ln�

1.781 · 3.318× 10−9a21

− 2 ln�

1.781 · 3.318× 10−9 b 21

��

= 5000− 7.52765�

ln�

5.90936× 10−9a21

− 2 ln�

5.90936× 10−9b 21

��

= 5000− 7.52765 ln

5.90936× 10−9a21

5.90936× 10−9b 21

�2

!

= 5000− 7.52765 ln�

1.6922312× 108 a21

b 41

= 4857.376+ 15.0553 ln�

b 21

a1

7. Solución: Se tienen los siguientes datos del reservorio:

k = 42 mDct = 1.8× 10−5 psi−1

Pi = 2727 psi

µ= 0.27 cPφ= 0.25h = 7.5 ft

B = 1.23 bbl/STB

Para los pozos se tiene que

Pozo A Pozo B Pozo CCaudal, q Observacional 200 STB/D 200 STB/DRadio, rw 0.33 ft 0.25 ft 0.375 ftSkin, s 5.0 7.5 1.5

a) Para calcular el delta de presión medido desde el pozo A con respecto al pozo B se tiene que

∆PAB =70.6qAµB

kh

Ei

−948φµct r 2

wA

ktA,1

+ 2sA

+70.6qAµB

kh

Ei

−948φµct r 2

wA

k�

tA,2− tA,1

+ 2sA

+70.6qB ,1µB

khEi

−948φµct r 2

AB

ktB ,1

+70.6

qB ,2− qB ,1

µB

khEi

−948φµct r 2

AB

k�

tB ,2− tB ,1

Dado que qA= 0 todo el tiempo, entonces

∆PAB =70.6µB

kh

qB ,1Ei

−948φµct r 2

AB

ktB ,1

+�

qB ,2− qB ,1

Ei

−948φµct r 2

AB

k�

tB ,2− tB ,1

��

NOTA: Hay una contradicción con respecto al dato de caudal de este pozo entre lo que se dice en el enunciado y latabla. Para efectos del ejercicio consideraremos lo dicho en el enunciado inicial.

Si reemplazamos los valores de los parámetros con los dados y operamos tenemos que

∆PAB = 0.0744326 [0+(100− 0)Ei (−0.5599125)] = 3.670 psi.

5

Page 6: Tarea01Resuelta

b ) Para calcular el delta de presión medido desde el pozo A con respecto al pozo C se tiene que

∆PAC =70.6qAµB

kh

Ei

−948φµct r 2

wA

ktA

+ 2sA

+70.6qCµB

khEi

−948φµct r 2

AC

ktC

=70.6qCµB

khEi

−948φµct r 2

AC

ktC

(esto dado que qA= 0 todo el tiempo)Si reemplazamos y operamos tenemos que

∆PAC =70.6qCµB

khEi

−948φµct r 2

AC

ktC

= 14.886514Ei (−0.921761) = 3.730 psi.

c) Como ya se evidenció en los dos literales anteriores, se tiene que∆PA= 0 (dado que qA= 0).

d ) PA= Pi − (∆PAB +∆PAC ) = 2727− (3.67+ 3.73) = 2719.6 psi.

8. Solución:

k = 440 mDct = 1.5× 10−5 psiPi = 1500 psi

µ= 0.27 cPφ= 0.26q1 = 250 STB/D

h = 60 ft

B = 1.3 bbl/STB

a) Tenemos que para t = 150 h,∆PW 2 = 1500− 1450= 50 psi=∆PW 1+∆PW I . En este orden de ideas

∆PW I = 50−∆PW 1, ⇒70.6qIµB

khEi

−948φµct

5002+ 4d 2�

kt

= 50−70.6q1µB

khEi

−948φµct r 2

1

kt

Nota: r 22 = r 2

1 +(2d )2 = 5002+ 4d 2.Al reemplazar por los valores dados y al operar se tiene que

0.234665Ei

−1.512491× 10−8 �5002+ 4d 2��= 50− 0.234665Ei (−0.0037812) = 50− 1.174326

Así que Ei

−1.512491× 10−8�

5002+ 4d 2��

= 208.06563.NOTA: Se infiere una inconsistencia en los datos proporcionados pues esta ecuación no posee solución. Losvalores entregados no justifican una caída de presión de 50 psi en el pozo 2.

b ) Debido a las inconsistencias encontradas en los valores dados este literal no fue desarrollado.

c) Debido a las inconsistencias encontradas en los valores dados este literal no fue desarrollado.

6